Open ES-1-Handout 2

Download as pdf or txt
Download as pdf or txt
You are on page 1of 61

1

Chapter 1 – GENERAL PRINCIPLES

Objectives:
At the end of the chapter, the students should be able to:
1. define mechanics and statics.
2. state the three Newton’s laws of motion.
3. memorize the units used in statics.

1.a. Definition of Terms


Mechanics – the branch of the physical sciences concerned with the state or motion of bodies that are subjected
to the action of forces.
Statics – division of mechanics of rigid bodies that deals with bodies which are acted on by balanced forces and
hence are either at rest or move with a constant velocity.
Length – a quantity that is needed to locate the position of a point in space and thereby describe the size of a
physical system.
Time – it is an absolute quantity that measures the succession of events.
Mass – a property of matter by which the action of one body with that of another can be compared. This
property manifests itself as a gravitational attraction between two bodies and provides a quantitative
measure of the resistance of matter to a change in velocity.
Force – it is the action of one body upon another body. The interaction can occur by direct contact or through a
distance by which the bodies are physically separated like the gravitational force.
Particle – a body that is thought of as a single point at which all its mass is concentrated.
Rigid body – a combination of a large number of particles in which all the particles remain at a fixed distance
from one another both before and after applying a load.
Concentrated force – represents the effect of a loading which is assumed to act at a point on the body.

1.b. Newton’s Laws of Motion

First Law
A particle originally at rest, or moving in a straight line with constant velocity, will remain in this state
provided the particle is not subjected to an unbalanced force.
F1

v
F3

F2

Second Law
A particle acted upon by an unbalanced force F experiences an acceleration a that has the same
direction as the force and a magnitude that is directly proportional to the force. If F is applied to a
particle of mass m, this law may be expressed mathematically as F = ma.
a
F
2

Third Law
The mutual forces of action and reaction between two particles are equal, opposite, and collinear.

F'

1.c. Weight
W = mg
where: g is the acceleration due to gravity (equals 9.81 m/s2 at standard location)

1.d. Units used in Mechanics

Quantity SI U.S.
Mass Kilogram Slug
Length Meter Feet
Time Second Second
2
Force Newton (kg-m/s ) Pound (slug-ft/s2)
3

Worksheet No. 1 – General Principles

NAME: _______________________________ DATE: ___________________

Answer the following:

1. What is mechanics?

2. Give the divisions of engineering mechanics and differentiate them.

3. Give the mathematical expressions for the Newton’s laws of motion.


4

Chapter 2 – FORCE VECTORS


Objectives:
At the end of the chapter, the students should be able to:
1. differentiate a scalar from a vector quantity.
2. add two forces using the Parallelogram Law.
3. resolve a force into components.
4. find the rectangular components of a force.
5. write a force in Cartesian vector form.
6. apply the dot product to find the rectangular component of a force along any direction.
7. use rectangular components to add concurrent forces.

2.a. Scalar and Vector Quantities


Scalar – a quantity that possesses a magnitude only such as mass, volume, length, and time.
Vector – a quantity that has both magnitude and direction. Examples of vector quantities are position, velocity,
acceleration, force and moment. It is represented by a bold letter, in print, or by a letter with an arrow
 
over it such as A , if it is handwritten. Its magnitude is designated by an italicized letter or A .

2.b. Multiplication and Division of a Vector by a Scalar


If a vector is multiplied by a positive scalar, its magnitude is increased by that amount. When multiplied
by a negative scalar it will also change the directional sense of the vector.

2A
A –A

2.c. Force
A force represents the action of one body on another and is generally characterized by its point of
application, its magnitude, and its direction.

2-d. Vector Addition. Finding a Resultant Force


Two vectors are added according to the parallelogram law. It states that “Two forces on a body can be
replaced by a single force called the resultant by drawing the diagonal of the parallelogram with sides
equivalent to the two forces.” The following procedure is to be followed in constructing the parallelogram:
1. Join the tails of the two forces at a point, say O, so that it makes them concurrent.
A
O
B

2. From the head of B, draw a line parallel to A. Draw another line from the head of A that is parallel to B.
These two lines intersect at point P to form the adjacent sides of a parallelogram.
A
O
B P

3. The diagonal of this parallelogram from O to P forms R, which then represents the resultant vector R = A +
B.
A
O  R
γ
B  P
5

The magnitude of the resultant R of two vectors is determined algebraically by applying the cosine law for a
general triangle. Thus,
R A2  B 2  2 AB cos 
The angles between the resultant R and vectors A and B can be determined by using the law of sine for a general
triangle. The resultant R has to lie always in between the given forces A and B, hence, γ should be chosen such
that 0 < γ <  .
An equivalent statement of the parallelogram law is the triangle law. Vector B is added to vector A by
connecting the head of A to the tail of B. The resultant R extends from the tail of A to the head of B.
A
B
R

As a special case, if the two vectors A and B are collinear, i.e., both have the same line of action, the
parallelogram law reduces to an algebraic or scalar addition R = A + B.
A B

R
2.e. Vector Subtraction
The resultant of the difference between two vectors A and B of the same type may be expressed as R' =
A – B = A + (–B), therefore, subtraction is defined as a special case of addition, so the rules of vector addition
also apply to vector subtraction.
P

R
–B
A
O
SAMPLE PROBLEMS
1. Determine the magnitude of the resultant of the two forces P and Q and the angle between this resultant and
the force Q.
P = 500 N
50°
30°
Q = 750 N
Solution: P = 500 N
50° R
a. Draw the parallelogram with P and Q as sides.
30°
Draw the diagonal representing R. 
Q = 750 N

b. The angle between the two forces P and Q is 80o, hence,  = 180º – 80 = 100. Applying the cosine
law, we have
R  500 2  750 2  2(500)(750) cos 100
R = 970.946 N
c. Applying the sine law, we get angle  between the resultant R and the force Q.
500 R

R sin  sin 
θ
α
P = 500 N 500 970.946

Q = 750 N
sin  sin 100
  30.47
6

d. Solving for , we have θ = 30.47º – 30º = 0.47.


Therefore, the resultant of P and Q is equal to 970.946 N at an angle of 0.47 counterclockwise
from the positive x-axis.

2. Two forces A and B, shown in the figure, have magnitudes of 50 kN and 80 kN, respectively. Determine a)
A + B; and b) 2A – B.

30
B
A 70
Solution:
a) Resultant of A + B
1. Draw the parallelogram whose sides are the given vectors.
80 kN
2. Draw the diagonal of the parallelogram from the tails of these R
vectors. This diagonal, R, represents the resultant of vectors A
and B.
3. Determine the angle opposite the resultant.
50 kN
 = 180  (70 + 30) = 80
4. Using the cosine law, find the magnitude of the resultant.
R2 = 802 + 502  2(80)(50) cos 80
R = 86.665 kN R
5. To find the angle of inclination of the resultant force, apply the θ
sine law. 30  80

R 80 50 kN 70kN
 30
sin β sin α
86.665 80
 ;  = 65.38
sin 80 sin α
Therefore,  = α – 30º = 35.38

b) Resultant of 2A – B
1. Draw the parallelogram whose sides are 2A and –B 2A = 100 kN
B = 80 kN
2. Draw the resultant force R
3. Compute the angle  R
 = 70 + 30 = 100.
4. Find the magnitude of R using cosine law.
R2 = 802 + 1002  2(80)(100) cos 100
R = 138.486 kN 80 kN
5. For the angle of inclination of R, apply sine law. 70
 R
138.486 80 30

sin 100 sin α 100 kN
α
θ
 = 34.67
and  = α + 30º = 64.67
7

3. Two forces having a magnitude of 1000 N and 600 N act on the ring. Determine the angle  between the
forces if the resultant force has a magnitude of 1400 N,.

600 N


1000 N

Solution:
a. Draw the parallelogram with the given forces as sides.
Draw the diagonal representing the resultant force. 600 N 1400 N
180° – 
b. Apply the cosine law to find the required angle. 
14002 = 10002 + 6002  2(1000)(600) cos (180  ) 1000 N

14002 = 1002 + 6002 + 2(1000)(600) cos


cos = 0.50
 = 60º

4. Determine the magnitude of the resultant force and its direction measured counterclockwise from the
positive x axis.
F1 = 250 N

1
52
45°

F2 = 400 N
Solution:
a. Determine the angle of inclination of F1.
θ1 = Arc tan 12 = 67.38º
5
b. Draw the parallelogram with F1 and F2 as sides.
Draw the diagonal representing R. F1 = 250 N
67.38°
c. Find the angle opposite R.
R
 = 180º – (67.38º + 45º) = 67.62 45°

d. Applying the cosine law, we have F2 = 400 N 

R  250  400  2(250)(400) cos 67.62


2 2

R = 382.558 N
e. Applying the sine law, we get angle  between the resultant R and F1.
250 R

sin  sin 
250 382.558 θ
 α
sin  sin 67.62 R

  37.18 F2 = 400 N 
F1 = 250 N
f. Solving for , we have θ = 45º – 37.18º = 7.82.
Therefore, the resultant is equal to 382.558 N at an angle of 7.82 clockwise from the positive x-
axis.
8

EXERCISES

1. Two forces act on the eyebolt. Determine the magnitude of the resultant force. Ans. 867.451 N
600 N

350 N
50°
20°
x

2. Determine the magnitude of the resultant force and its direction measured counterclockwise from the
positive x axis. Ans. 1322.876 N, 49.11º
y
1500 N

x
30°

1000 N

3. The wires AB and AD help support pole AC. Knowing that the tension is 240 lb in AB and 80 lb in AD, find
the magnitude and direction of the resultant of the forces exerted by the wires at A. Ans. 278.155 lb at
66.98º from the horizontal
A
·
10 ft

8 ft C 6 ft D

4. Determine the magnitude of the resultant force and its direction measured from the positive x-axis. Ans. 5.75
kN, 113.02º y

x
3
4 30
3.5 kN
5 kN
5. For the given forces F1 and F2, determine the magnitude of their difference F' = F1 – F2 and the angle θ
which F' makes with the positive x axis. Ans. 25.608 kN, 1.47º
y
F2 = 16 kN F1 = 22 kN

3
60 4
x

6. Two forces are applied as shown to a hook support. Determine the required angle  if the resultant R of the
two forces applied to the support is to be horizontal. What is the magnitude of the resultant?
70 N
30º
α

45 N
9

Worksheet No. 2 – Scalar and vector quantities and vector addition

NAME: _______________________________ DATE: ___________________

I. Are the following quantities vectors or scalars? Explain.

1. The population of the world.

2. The current in a river.

3. The cost of purchasing a new bag.

4. Your path going to school.

5. The power consumption in your household.

6. Your body temperature.

7. How long it takes to do a mathematics assignment.

8. The magnitude of the force required to push a trash can.

9. The acceleration of a stone dropped into a well.

10. The torque required to rotate a shaft.

II. Addition of vectors

1. A force F1 has a magnitude of 20 kN and a direction of 0°. Force F2 has a magnitude of 40 kN and a
direction of 60°. Find the magnitude and direction of the resultant force to the nearest whole number.
F2

60°
F1
10

2. Find the magnitude of the sum R = A + B of the given forces. What is the direction of R measured from
the positive x axis?
y

A = 5 kN
20°
x
75°

B = 5 kN

3. Find the resultant R = A – B of the forces given as shown. Also find the direction angle measured form
the positive x axis.
A = 8 kN y
B = 4.5 kN

45° 60°
x
11

2.f. Finding the Components of a Force


Components – two separate forces equal to a single force.
Resolution – it is the process of replacing a force with its components.
In order to determine the magnitude of each component of the force F along the a and b axes, the
following procedure is applied:
1. Construct a parallelogram by drawing lines starting from the tip of the force, a line parallel to a, and a line
parallel to b, intersecting the a and b axes.
2. Label the sides of the parallelogram as the force components Fa and Fb.
3. Apply the law of sines to one of the triangles to determine the unknown magnitudes of the components.
b b

F F
Fb
O O
a Fa a

SAMPLE PROBLEMS
1. Resolve the 800-kN force into two components: one along the y-axis and the other along the x’-axis.
y
800 kN

45
20
x
x
Solution: ’
a. Draw lines parallel to the given axes from the tip
of the given force to form a parallelogram.
y

F = 800 kN
Fy 
Fy
 
Fx’
x’

b. Determine the interior angles of one triangle.


 = 45° + 20° = 65
 = 90° – 20° = 70
 = 90° – 45° = 45.

c. Applying sine law, find the required components.


F F Fy
 x' 
sin α sin β sin θ
800 Fx' Fy
 
sin 70 sin 45 sin 65
Fx’ = 601.990 kN and Fy = 771.578 kN
12

2. Determine the magnitudes of the components of the 80-lb force if it is resolved into components acting
along the a and b axes.
b

45
a
120

80
lb
Solution:
a. Extend the b-axis downward and the a-axis to the left.
From the tip of the force, draw lines parallel to the axes
to form the parallelogram.
b

Fa a
Fb

 
80 lb
b. Solve for the interior angles
 = 45
 = 180° – 120° = 60
 = 120° – 45° = 75
c. Apply sine law
80 Fa Fb
 
sin 45 sin 75 sin 60
Fa = 109.282 lb and Fb = 97.980 lb

3. The 500-N force is to be resolved into components along lines a-a’ and b-b’. If its component along a-a’ is
to be 375 N, determine the angle  and the corresponding value of the component along b-b’.
a
b
500 N
45 

a’ b’
a
Solution:
a. From the tip of the force, draw lines parallel to a-a’ and b-b’
to form a parallelogram. 45
b. Applying sine law, compute for . 375 N   500 N

375 500
 ,  = 32.03
sin β sin 45
Fb
c. Computing for α, we have  = 180° – (45° + 32.03°) = 102.97
d. Compute for Fb using sine law
Fb 500 b’
 , Fb = 689.067 N
sin α sin 45
13

4. It is required that the resultant force acting on the eyebolt be directed along the positive x axis and that F2
have a minimum magnitude. Determine this magnitude, the angle θ, and the corresponding resultant force.
(Hibbeler, 2010)
F1 = 800 N

60º

F2
Solution:
a. Using the triangle rule, draw the forces.

F2
F1 = 800 N
60º θ

b. Since the magnitudes of F2 and R are unknown, F2 can be any vector that has its head touching the
line of action of R. F2 will be minimum when its line of action is perpendicular to R or when θ = 90º
(by inspection of the triangle drawn).

F1 = 800
F2
N
60º

R
c. Using the right triangle, solve for R and F2.
R  800cos 60 = 400 N
F2  800 sin 60 = 692.820 N

Alternate solution:
From sine law:
F2 F2 800
F1 = 800 N

60º θ sin 60 sin 
R 800 sin 60
F2 
sin 
692.82
F2 
sin 
dF2
F2 will be minimum if  0 ; hence,
d
dF2 sin  (0)  692.82 cos
 0
d sin 2 
From which cos θ = 0 or θ = 90º.
14

EXERCISES

1. Find the components of the 1500-N force shown acting along the u and v axes.
Ans. 1326.828 N,1627.595 N

45°
u
70° 60°

v
1500 N

2. The 320-lb force is to be resolved into components acting along the u and v axes. Determine the magnitudes
of these components. Ans. 490.268 lb, 431.135 lb
v

40°
u
120°

320 lb

3. Determine the components of the force F = 10 kN in the directions of the x’and y axes. Ans.16 kN, 7.856 kN
y x'

y'
F
60 3
30 4 y
y x

4. The 450-lb force is to be resolved into components acting along members AB and AC of the frame shown.
Determine the magnitude of each component. (Hibbeler, 2010) Ans. 869.333 lb, 636.396 lb

A 30º

C
45º

450 lb

B
15

5. The resultant of the two forces shown is 500 lb directed to the right along the x-axis. Find the magnitude and
direction of F. Ans. 433.128 lb, 28.68º
y

θ
x
60º

240 lb

6. The vertical force with magnitude 75 lb acts downward at A on the two-member frame shown in the figure.
Determine the magnitudes of the two components of the force directed along members AB and AC.
Ans. 67.243 lb, 54.904 lb
B

45º

30º
75 lb
C

7. The gusset plate is subjected to the two forces shown. Replace these forces with two equivalent forces; Fx in
the x direction and Fa in the a direction. What are the magnitudes of Fx and Fa?
Ans. –720.644 N, 1465.923 N

A x
10º
750 N
1000 N 45º
25º
a
16

Worksheet No. 3 – Force Components

NAME: _______________________________ DATE: ___________________

Solve each of the following problems.

1. Determine the components of the 1500-lb force F along the oblique axes a and b.
F = 1500 lb

b
70°
a
35°

2. Resolve the 3.5-kN force along the oblique axes u and v.


v

F = 3.5 kN

120°
45°
u
17

3. The 300-lb force is to be resolved into components along lines a-a' and b-b'. If its component along line a-a'
is to be 240 lb, determine the angle θ. Also find the corresponding value of the component along b-b'.

a a'
60°
θ

b 300 lb
'

4. Resolve the 750-N force into components acting along the x and y’ axes.
y' 750
y N
x
30º 35º

45º
18

2.g. Rectangular Components of a Force


Rectangular components of a force – components that are perpendicular with one another. These components
can be determined from trigonometry.
y
Fx = F cos  ,

Fy
F Fy = F sin 

x
Fx
The direction of F can also be defined using the slope of its line of action. The slope triangle and the
force triangle are similar, hence, proportional. The components of the force can then be determined using ratio
and proportion.
F Fx Fy F
d   ; where: d  x 2  y 2 `
Fy y
x y d
x
x y
Fx Fx  F Fy  F
d d

SAMPLE PROBLEMS

1. Determine the x and y components of each of the forces shown.


y
F1 = 120 N
F4 = 150 N
7
55
0 70
x
5
1
112.5 mm F2 = 390 N
2 N
F3 = 221

60 mm
Solution:
a. Components of F1
y
F1 = 120 N
From trigonometry,
F1x = 120 cos 70 F1y = 120 sin 70
F1y = 41.042 N = 112.763 N
70
x
F1x

b. Components of F2
y Find the hypotenuse of the slope triangle
F2x d  12 2  52 = 13
x
F2y 5
12
From the similarity of triangles,
F2 = 390 N 12 5
F2 x  (390) F2 y  (390)
13 13
= 360 N = 150 N
19

c. Components of F3
y Find the hypotenuse of the slope triangle

x
d  602  112.52 = 17
F3y From the similarity of triangles,
F3 = 221 N
112.5 mm 8 15
F3x F3 x  (221) F3 y  (221)
17 17
60 mm = 104 N = 195 N

d. Components of F4

y From trigonometry,
F4x = 150 sin 55 F4y = 150 cos 55
F4 = 150 N
= 122. 873 N = 86.036 N
55 F4y
70

x
F4x

2. Find a) the x and y components, and b) the x´ and y´ components of each of the forces shown.
y
F1 = 800 lb

25
45
x
30

F2 = 650 lb

Solution:
a. For F1
y y From trigonometry:
F1 = 800 lb F1 = 800 lb F1x  800 cos 70  273.616 lb
F1y y´ y´ F1 y  800 sin 70  751.754 lb
25 25 F 1y'
F1x  800 sin 25  338.095 lb
45 F1x' 45
x x F1 y  800 cos 25  725.046 lb
F1x

b. For F2 x´ x´
y y From trigonometry:
F2 x  650 cos 30  562.917 lb
y´ y´
F2 y  650 sin 30  325 lb
F2y' F2 x  650 sin 75  627.852 lb
45 F 45
2x
x x F2 y  650 cos 75  168.232 lb
30 30
F2y
F2 = 650 lb F1x' F2 = 650 lb
x´ x´
20

3. The box on the inclined plane has a mass of 40 kg. Find the components of the weight of the box that are
parallel and perpendicular to the incline.
y y
x
Wx

25 W = 392.4 N
25 Wy
x
Solution:
a. Determine the weight of the box.
W = 40(9.81) = 392.4 N
b. From the tip of the weight, draw a line along the y-axis and from the tail of the weight, draw a line
parallel to the x-axis. The triangle formed should follow the triangle law.
c. Find the required components of the weight.
Wx = W sin 20, Wx = 392.4 sin 25, Wx = 165.835 N
Wy = W cos 20, Wy = 392.4 cos 25, Wy = 355.635 N

4. Determine the components of the force acting on the box shown along the x-y axes which are parallel and
perpendicular to the incline.
y
F = 800 N
3 x
4

5
12

Solution:

a. Find the angle of inclination of F and the angle the inclined plane makes with the horizontal.
F = 800 N
y
α1 = Arc tan 3 = 36.87º
x 4
α1
α2 α2 = Arc tan 5 = 22.62º
12

α2 θ = α1 + α2 = 59.49º

b. Use trigonometry to find the required components.


Fx
y
θ Fy Fx = 800 cos 59.49
F = 800 N x
θ = 59.49º = 406.151 N
Fy = 800 sin 59.49
= 689.232 N
21

EXERCISES
1. Find a) the x and y components and b) the x´ and y´ components of each of the forces shown. Ans. a) For F1,
–1359.462 N and 633.927 N; for F2, 1697.056 N and 1697.056 N, b) For F1, –1149.067 N and 964.181 N;
for F2, 2318.222 N and 621.166 N
y

F2 = 2400 N
F1 = 1500 N x´
45
25 30 x

2. Determine the components of the force along the x-y axes which are parallel and perpendicular to the
incline. Ans. 156.498 lb, 313.063 lb
y

P = 350 lb
x
1
2

3
4

3. The force P that is applied to the wagon, as shown in the figure, can be resolved into two components: one
parallel and one perpendicular to the incline. To pull the wagon up the incline, the parallel component must
be at least 1000 sin  N. If P = 320 N, determine the slope angle  of the steepest incline that can be
ascended. (Pytel, 1996) Ans. 16.28º
P

1
1

4. The x-component of the force P is equal to 450 N. What is the magnitude of the force and the corresponding
y-component? Ans. 522.321 N, 265.177 N
y P

5 x
4 12
3 3
4

O
5. In the figure shown, the cable AB prevents bar OA from rotating clockwise about the pivot O. Determine the
n and t components of this force acting at point A of the bar if the cable tension is 1200 N. Ans. 512.560 N,
1085.024 N
t
n
A

2,0 m

O 60
B 
1.5 m
22

2.h. Rectangular Components of a Force in Three Dimensions


The rectangular components for any force, as discussed in the previous section, can be found from
trigonometry. z
z

z
x
x Fy F y
y
x FFx = F cos 
x x x
z
z z

F Fz z
F
y
y y
Fy

x Fy = F cos y x Fz = F cos z
z
If the rectangular components are known, the magnitude of F can be determined from the relationship
2 2 2
F  Fx  Fy  Fz
Direction angles or direction cosines – these are the angles x, y, and z, (where 0    180) between F and
the positive coordinate axes which can be computed as follows:
Fx Fy Fz
x = Arc cos , y = Arc cos , z = Arc cos
F F F
The cosines of these angles must satisfy the equation
` cos2 x + cos2 y + cos2 z = 1
If an angle is more than 90, the cosine is negative, indicating that the sense of the component is opposite to the
positive direction of the coordinate axis.

2.i. Cartesian Vectors


The analysis of some problems in three dimensions is simplified using vector algebra in which the
forces are represented in the Cartesian vector form. A right-handed coordinate system is used in the theory of
vector algebra. A rectangular or Cartesian Coordinate System is said to be right-handed if the thumb of the right
hand points in the direction of the positive z axis when the right-hand fingers are curled about the axis directed
from the positive x to the positive y axis.

2.i-1. Unit vectors


Unit vector, uA – a vector with a magnitude of one “dimensionless” unit and acts in the same direction as the
vector A. It is determined by dividing A by its magnitude A; i. e.,
A z
uA 
A
2.i-2. Cartesian unit vectors k j
The directions of the positive x, y, and z axes are defined by the y
i
Cartesian unit vectors i, j, k , respectively.
x
23

2.i-3. Cartesian vector representation


Using the properties of vector addition, a vector A can be written as the sum of its three components:
A = Ax i + Ay j + Az k
This is known as the standard Cartesian form of representing a vector. Hence, for a force F with a given
magnitude and direction cosines, the Cartesian vector form is
F = F cos x i + F cos y j + F cos z k

2.j. Position Vectors


Position vector – it is defined as a fixed vector which locates a point in space relative to another point.
If r extends from the origin of coordinates O to point P(x, y, z), then r can be expressed in Cartesian
vector form as r = xi + yj + zk.
z

P(x, y, z)
zk r
O yj
xi
B y

For a position vector r that is directed from point A to point B in space, applying the head-to-tail vector
addition and using the triangle rule, rA + r = rB. Solving for r and expressing rA and rB in Cartesian vector form
yields
r = rB – rA = (xBi + yBj + zBk) – (xAi + yAj + zAk)
or r = (xB – xA)i + (yB – yA)j + (zB – zA)k
z B(xB, yB, zB)
r
rB
A(xA, yA, zA) y
rA

x
Thus, the i, j, k components of the position vector r may be formed by taking the coordinates of the tail of the
vector A(xA, yA, zA) and subtracting them from the corresponding coordinates of the head B(xB, yB, zB).

2.k. Force Vector Along a Line


A force F with a line of action that is directed from point a A to a point B can be expressed as a
Cartesian vector by realizing that it has the same direction and sense as a position vector r acting along its line
of action. This common direction is specified by the unit vector u. Hence, from the definition of a unit vector,
F = F u; where: u  r
r
r
FF
r

FF
xB  x A i   y B  y A j  z B  z A k
xB  x A 2   y B  y A 2  z B  z A 2
24

SAMPLE PROBLEMS

1. The slope of the 7.5-kN force is specified as shown in the figure. Express F in Cartesian vector form.
y

F = 7.5 kN
5
12
x

Solution:
a. Determine the hypotenuse of the slope triangle
d  12 2  52 = 13
b. Find the x and y components of the given force
Fx = 12 (7.5) = 6.923 kN to the left
13
5
Fy = (7.5) = 2.885 kN downward
13
c. Write the force in Cartesian vector form
F = –6.923i – 2.885j

2. The line of action of the 2500-lb force is directed from point A to point B as shown in the figure. Determine
the x and y components of the force and express it in Cartesian vector form.
y, m

B(8, 6)

F = 2500 lb
x, m
O
A(–7, –2)

Solution:
a. Write the position vector r acting along AB in Cartesian form
r = [8 – (–7)] i + [(6 – (–2)] j ; r = 15i + 8j
b. Determine the magnitude of r
r  152  82 = 17
c. Write F in Cartesian vector form
r
FF = (2500) 15i  8 j
r 17
F = 2205.882i + 1176.471j
d. Hence, Fx = 2205.882 lb and Fy = 1176.471 lb
25

3. Find the rectangular representation of the force F shown given that its magnitude is 750 N and determine the
unit vector acting along its line of action.
z
6m
4m
A
F
3m
B y

Solution:
a. Find d (length of AB).
d  42  62  32 = 7.81
b. Express the force in Cartesian vector form.
F  F u AB

F  750
  4ˆi  6ˆj  3kˆ 
7.81
F = 384.123î + 576.184ĵ 288.092 k̂
c. Determine the unit vector acting along F
 4i  6 j  3k
u AB 
7.81
uAB = –0.512i + 0.768j – 0.384k

4. The force F, directed along the line AB, has a magnitude of 8 kN. Determine the rectangular representation
of F and the unit vector directed from A to B.
y
8m
AF
A a
10 m
4m
6m B x
A
z
A
Solution:
a. From the figure, the components of the position vector, a, acting from A to B are
x = 6, y = 10, and z = 12.
b. Writing a in rectangular form,
a = 6i 10j 12k.
c. Find the magnitude of a
a  6 2  10 2  12 2 = 16.733
a
c. The unit vector acting along AB is u  .
a
6i  10 j  12k
u = 0.359i 0.598j 0.717k
16.733
d. The rectangular representation of F is F = Fu. Hence,
F = 8 (0.359i 0.598j 0.717k) = 2.872i 4.784j 5.736k
26

5. Express the force as a Cartesian vector.


z F = 250 N
A

O 45º
5
4 3
B
C
x y

Solution:
a. In triangle AOB,
AB = 250 sin 45º = 176.777 lb; OB = 250 cos 45º = 176.777 lb
b. In triangle BOC,
4 3
OC  (176.777 ) = 141.422 lb; BC  (176.777 ) = 106.066 lb
5 5
c. The components of F are
Fx = –106.066 lb, Fy = 141.422 lb, and Fz = 176.777 lb
Hence,
F = 106.066i + 141.422j + 176.777k

EXERCISES

1. The magnitude of force F is 250 lb. Express F in Cartesian vector form. Ans. F = 216.506i 125j lb
y, ft

F
x, ft
O 30º
A(4, –2)

2. Express the force as a Cartesian vector. Ans. F 247.487i 428.661j +494.975k N


F = 700 N z

45º
O
60º

y
x
3. Express the position vector rAB in Cartesian vector form. Ans. rAB = –6i +6j +3k
z B
3m
rAB
3m
y
3m 4m

A
2m
x
27

4. The force F has a magnitude of 80 lb and acts at the midpoint C of the thin rod. Express the force as a
Cartesian vector. (Hibbeler, 1986) Ans. F = 34.286i + 22.857j 68.571k lb
z

6 ft
C
F

y
O
3 ft
A

2 ft

5. Express the force F as a Cartesian vector. Ans. F = 126.566i +253.133j 506.266k N


z

3m

3m

5 y
x 4m m

F = 580 N 5m
B

6. The cable AB exerts a tension of 2.5 kN on the fixed bracket at A. Write the vector expression for the vector
T. (Meriam, 2002) Ans. F = 2.301i + 0.959j + 0.192k kN
z
0.5 m
B
0.4 m
y

T = 2.5 kN

0.3 m
1.2 m
x
28

2.l. Dot Product or Scalar Product


Dot or scalar product of two intersecting vectors – it is defined to be a scalar quantity determined by
multiplying the magnitudes of the two vectors by the cosine of their included angle.
A
A · B = AB cos 

where: 0    180
B

For 0    90, the scalar is positive and for 90    180, the scalar is negative. When  = 90,
the two vectors are perpendicular and the scalar is zero.
One use of the dot product is to obtain the rectangular component of one vector along any direction.
This component is equal to the dot product of the vector with a unit vector in the desired direction. From the
figure,
A A · B = AB cos 
AB
 A cos θ 
AB B
B AB = A · n B

2.l-1. Properties of the dot product:


1. Commutative: A · B = B · A
2. Associative: mA · nB = mnA · B
3. Distributive: A · (B + C) = A · C + A · C

2.l-2. Dot product of the unit vectors


From the definition of a dot product, the dot products of the orthogonal unit vectors are as follows:
i · i = 1×1×cos 0º = 1 j · j = 1×1×cos 0º = 1 k · k = 1×1×cos 0º = 1
i · j = 1×1×sin 0º = 0 j ·i = j ·k = k ·j = k ·i = i ·k = 0

2.l-3. Dot product of two vectors


A · B = (Ax i + Ay j + Az k) · (Bx i + By j + Bz k)
The vector multiplication can be evaluated by applying the properties of a dot product.
A · B = Ax i + Ay j + Az k · i + By j + Bz k
= Ax Bx i · i + Ax By i · j + Ax Bz i · k + Ay Bx j · i + Ay By j · j
+ Ay Bz j · k + Az Bx k · i + Az By k · j + Az Bz k · k
Applying the dot product operations, we finally get
A · B = AxBx + AyBy + AzBz
From the above result, the expansion of a dot product in terms of the scalar components of the vectors is
equal to the sum of the products of their respective scalar components.
29

SAMPLE PROBLEMS
1. Find the component of the force A = {5i + 2j – 3k} kN that is parallel to B = {–4i + 3k} kN.
Solution:
a. Find the unit vector acting along the line of action of B
B  4i  3k
nB   = – 0.8i + 0.6k
B 4 2  32
b. Find the component of A parallel to B
AB = A · n B
= (5i + 2j  3k) · (– 0.8i + 0.6k)
= – 4 – 1.8
= –5.8 kN
The negative sign indicates that the component of A along B is opposite the direction of B

2. Resolve A = {6i  4j + 5k} kN into two vector components – one parallel to and the other perpendicular to
B = {4î + 2k} kN. Express each of your answers as a vector.

Solution:
a. Find the component of A parallel to B. Denote this component as At.
4i  2k
At = A · n B = (6i  4j + 5k)
42  22
6( 4)  5( 2)
= = 7.603 kN
20
b. Express At in Cartesian vector form
4i  2k
At = At n B = 7.603
42  22
= 7.603(0.894i + 0.447k)
At = 6.797i + 3.399k kN
c. The component of A perpendicular to B, denoted as An, is
A = At + An ; An = A – At
An = (6i  4j + 5k)  (6.797i + 3.399k)
= (3 – 6.797)i – 4j + (5 + 3.399)k
An = –3.797i  4j + 8.399k kN

3. Using the dot product, find the components of the force F = {2î + ĵ} kN in the directions of the x' and y'
axes.
y x'

y'
y
x'
30 60
x

y' ux’
Solution: uy’
a. Determine the unit vectors acting along the x' and y' axes 30
60
x
ux’ = cos 60º i + sin 60º j = 0.5i + 0.866j
30

uy’ = cos 30º i + sin 30º j = 0.866i + 0.5j

b. Find the required components y


x'
Fx’ = F · ux’ = (2i + j) · (0.5i + 0.866j)
= 2(0.5) + 1(0.866) y'
= 1.866 kN Fx’

Fy’ = F · uy’ = (2i + j) · (0.866i + 0.5j) 60


30
x
= 2(0.866) + 1(0.5)
Fy’
= 1.232 kN

4. In the system shown, a force F acts from B to D. Find the magnitude F if its component along the the boom
ABC is 850 lb.
E
y

3’
6’

D 6’
8’
4’ A
3’
z x
9’
B

C
Solution:

Point x coordinate, ft y coordinate, ft z coordinate, ft


B 8 –3 0

D 0 0 6

C 0 –9 0

a. Find the unit vector acting from B to C.


 8i  6 j  8i  6 j
n BC  = =  0.8i  0.6j
8 6
2 2 10
b. Express F in Cartesian vector form
 8i  3 j  6k  8i  3j  6k
F = F nBD = F =F
8 3 6
2 2 2
109
c. Determine the magnitude of F using dot product
FBC = F · n BC

 8i  3 j  6k
850 = F · ( 0.8i  0.6j)
109
6.4  1.8
850 = F
109
F = 1929.187 lb
31

5. The force FAB = 800 N acts along cable AB of the structure shown. Determine the projected component of
the force along cable AC. Express the result as a Cartesian vector.
z

1.5 m
B
1.5 m
C
1m

3m
FAB = 800 N
O A
4m y
x

Solution:

Point x coordinate, m y coordinate, m z coordinate, m

A 0 4 0

B –1.5 0 1
C 1.5 0 3

a. Express the force in Cartesian vector form


 1.5i  4 j  k  1.5i  4 j  k
F = F nAB = 800 = 800
1.5 2  4 2  12 4.387
= 273.505i – 729.348j + 182.337k
b. Find the unit vector acting from A to C
1.5i  4 j  3k 1.5i  4 j  3k
n AC  =
1.5  4  3
2 2 2 5.22

= 0.287i  0.912j + 0.684k


c. Find the required component
FAC = F · nAC = (273.505i – 729.348j + 182.337k) · (0.287i  0.912j + 0.684k)
= –273.505(0.287) – 729.348(– 0.912) + 182.337(0.684)
FAC = 711.388 N
d. Express FAC in Cartesian vector form
FAC = FAC nAC = 711.388(0.287i  0.912j + 0.684k)
FAC = 204.168i – 648.786j + 486.589k
32

EXERCISES
1. Determine the magnitude of the component of the position vector r along the Oa axis. Ans. 8 ft
z

6 ft
r
a
12 ft

18 ft
O 6 ft
y
12 ft
x

2. Two forces are applied at a point in a body as shown in the figure. Determine the magnitude of the
rectangular component of force F1 along the line of action of force F2. (Riley, 1993) Ans. 30.53 lb
z
B

A
F1 = 120 lb
F2 = 150 lb
60 4.5 ft

O
2 ft
6 ft y
1.5 ft
x

3. In the system shown, a force F acts from B to D. Find the magnitude of F if its component along line AC is
equal to 1200 lb. Ans. 4474.417 lb

y
E
3’
6’
6’
D 8’
A
4’
z x
9’ 3
’ B

4. Determine the magnitudes of the components of force F = 56 N acting along and perpendicular to line AO.
(Hibbeler, 2010) Ans. 46.9 N; 30.7 N
33

Worksheet No. 4 – Rectangular Components

NAME: _______________________________ DATE: ___________________

Work on the following problems.

1. Determine the x and y components of each of the forces shown.


y y
y
F = 1200 lb
F = 1000 N 15
x
30 8

60 F = 920 lb
x x
(a) (b) (c)

2. Resolve the force F into two components, one acting parallel and the other acting perpendicular to the u
axis.

3. A force P directed along line BD is exerted on member ABC by member BD. If P must have a 300-lb
horizontal component, determine the magnitude of the force P and its vertical component.

D C

40º

F
34

4. Determine the components of F = 275 lb that act along rod AC and perpendicular to it. Point B is located at
the midpoint of the rod.
z

6 ft
B

y
O
F 3ft
6ft A

2ft
2 ft
C
x

5. Find the components of the given force and the direction angles with respect to the positive reference axes.
z
A

60
F = 600 lb 3 ft
y

5 ft
A

B 1.5 ft
A A x
35

2.m. Component Method of Adding Forces. Resultant of Concurrent Forces


When three or more forces are to be added, an analytic solution of the problem can be obtained by
resolving each force into their rectangular components. For three forces F1, F2, and F3 acting on a particle, their
resultant R is defined by the relation
R = F1 + F2 + F3
Resolving each force into its rectangular components,
Rxi + Ryj + Rzk = F1xi + F1yj + F1zk + F2xi + F2yj + F2zk + F3xi + F3yj + F3zk
Rxi + Ryj + Rzk = (F1x + F2x + F3x)i + (F1y + F2y + F3y)j + (F1z + F2z + F3z)k
from which it follows that
Rx = F1x + F2x + F3x Ry = F1y + F2y + F3y Rz = F1z + F2z + F3z
or Rx = Fx Ry = Fy Rz = Fz
Therefore, the scalar components Rx, Ry, and Rz of the resultant R of several forces acting on a particle are
obtained by adding algebraically the corresponding scalar components of the given forces.
The magnitude and direction angles of the resultant R can be determined from the relations
2 2 2
R  Rx  R y  R z
Rx Ry Rz
x = Arc cos y = Arc cos z = Arc cos
R R R
For two dimensional forces, the two components are
Rx = Fx Ry = Fy
The magnitude of R is found from Pythagorean theorem; that is,
2 2
R  Rx  R y
The angle which specifies the direction of the resultant force measured from the reference x-axis is determined
from trigonometry; i. e.,
Ry
 = tan–1
Rx

SAMPLE PROBLEMS
1. Find the resultant of the given system of forces using the component method.
y

88 lb
65 lb 75
35
x
45

125 lb

Solution:
a. Resolve each force into x and y components.

Force x - component y - component


65 lb  65 sin (75º) =  62.785 lb 65 cos (75º) = 16.823 lb
88 lb 88 cos (35º) = 72.085 lb 88 sin (35º) = 50.475 lb
125 lb 125 cos (45º) = 88.388 lb  125 sin (45º) =  88.388 lb
36

b. Summing up the x and y components gives:


Rx = 62.785 + 72.085 + 88.388 = 97.688 lb
Ry = 16.823 + 50.475  88.388 =  21.09 lb
c. The magnitude of the resultant R yields

R = 97.6882  21.09 2 = 99.939 lb


d. The direction angle of the resultant force, from trigonometry, is
Rx 21.09
 = tan–1 = 12.18º
Ry θ 97.688

2. Three ropes are tied to a small metal ring. At the end of each rope three students are pulling, each trying to
move the ring in their direction. Find the net force (magnitude and direction) on the ring due to the three
applied forces.
y
B = 70 lb
A = 25 lb
135
37
x
240

C = 60 lb

Solution:
a. Resolve the forces into x and y components.

Force x – component, lbs y – component, lbs

A = 30 lb Ax = 25 cos 37o = +19.966 Ay = 25 sin 37o = +15.045

B = 50 lb Bx = 70 cos135o = – 49.497 By = 70 sin135o = +49.497

C = 80 lb Cx = 60 cos 240o = –30.0 Cy = 60 sin 240o = –51.962

Total Rx = – 59.531 Ry = 12.58 lbs

b. Find the magnitude and direction angle of the resultant force.


Rx
x R = 59.5312  12.582 = 60.846 lb
θ
12.58
Ry  = tan–1 = 11.93º
59.531
c. Thus the resultant force on the ring is 60.846 pounds acting at an angle of 191.93 from the positive
x axis.
37

3. Three forces act on the bracket as shown. Determine the magnitude and direction  of F3 so that the
resultant force is directed along the positive x' axis and has a magnitude of 1000 N. (Hibbeler, 1986)

y y'
F1 = 450 N

45
F2 = 200 N
x
30

x'

F3
Solution:
a. Determine the x' and y' components of the given forces and the resultant.
F1x’ = 450 cos 75 = 116.469 ; F1y’ = 450 sin 75 = 434.667
F2x’ = 200 cos 30 = 173.205 ; F2y’ = 200 sin 30 = 100
Rx’ = 1000 ; Ry’ = 0
b. Find the components of F3 along the x' and y' axes
Rx' = F1x' + F2x' + F3x'
1000 = 116.469 + 173.205 + F3x'; F3x' = 710.326 N
Ry' = F1y' + F2y' – F3y'
0 = 434.667 + 100 – F3y' ; F3y' = 534.667 N
c. Compute for the magnitude and direction angle of F3
F3  710.326 2  534.667 2 = 889.062 N
534.667
 = Arc tan = 36.97
710.326

4. Determine the magnitude of the force F so that the resultant force R of the three forces is as small as
possible. What is the minimum magnitude of R? (Hibbeler, 1986)
y

5 kN
x

30

4 kN F

Solution:
a. Determine the components of R
Rx = Fx = 5 – F sin 30 = 5 – 0.5F
Ry = Fy = F cos 30  4 = 0.866F  4
b. Express R as a function of F
R2 = Rx2 + Ry2
R2 = (5 – 0.5F)2 + (0.866F  4)2
38

c. Find the derivative of R with respect to F and equate to zero


dR
2R  2(5  0.5 F )(0.5)  2(0.866 F  4)(0.866)
dF
0 = 5 + 0.5F + 1.5F – 6.928
0 = 2F – 11.928
F = 5.964 kN
d. Find the minimum value of R
R2 = [5 – 0.5(5.964)]2 + [0.866(5.964)  4]2
R = 2.33 kN

5. Determine the magnitude of the resultant force and the angles between the line of action of the resultant and
the reference axes.
z
F2 = 18 kN

40 F3 = 25 kN
F1 = 12 kN
35

26 50
42 60 y

x
z
Solution:
z
z F3 = 25 kN
F1 = 12 kN
F2 = 18 kN F3y
50 F3z
F1z 26 y 60 y
42 F1x F2z F3x
F1y 40 x
F2x
35
x
F2y y
x

a. Find the components of the given forces

Force x-component, kN y-component, kN z-component, kN

F1 12 cos 26 cos 42 = 8.015 –12 cos 26 sin 42 = –7.217 12 sin 26 = 5.260

F2 –18 cos 40 sin 35 = –7.909 –18 cos 40 cos 35 = –11.295 18 sin 40 = 11.570
F3 25 cos 50 cos 60 = 8.035 25 cos 50 sin 60 = 13.917 25 sin 50 = 19.151
Total Rx = 8.141 Ry = –4.595 Rz = 35.981

b. Find the magnitude of the resultant force

R= 8.1412  4.595 2  35.9812 = 37.176 kN


39

c. Determine the direction angles


z
8.141
x = Arc cos = 77.35
37.176
 4.595
Rz y = Arc cos = 97.1
R 37.176
z
35.981
Ry y z = Arc cos = 14.57
37.176
x y
Rx
x

6. Three forces, F1 = 136 N, F2 = 250 N, and F3 = 325 N, are applied with cables to the anchor block shown.
Determine the magnitude and direction angles of the resultant of the three forces. (Riley, 1993)
z
0.6 m
A
B
F1
2.7 m
F2
3.6 m
1.8 m
F3  2.7 m
y
C 3.6 m 2.4 m
0.9 m
1.2 m
x

Solution:
Force Position Vector x, m y, m z, m d, m

F1 OA 2.4 2.7 3.6 5.1


F2 OB 0.6 –1.8 2.7 3.3
F3 OC 3.6 –1.2 0.9 3.9

a. Write the forces in Cartesian vector form


2.4i  2.7 j  3.6k
F1 = 136 = 64i + 72j + 96k
5 .1
0.6i  1.8 j  2.7k
F2 = 250 = 45.455i  136.364j + 204.545k
3 .3
3.6i  1.2 j  0.9k
F3 = 325 = 300i 100j + 75k
3.9
b. Find the magnitude of the resultant force
Rxi + Ryj + Rzk = 64i + 72j + 96k + 45.455i  136.364j + 204.545k + 300i 100j + 75k
Rxi + Ryj + Rzk = (64 + 45.455 + 300)i + (72  136.364 100)j + (96 + 204.545 + 75)k
Rxi + Ryj + Rzk = 409.455i – 164.364j + 375.545k

R = 409.455  164.364  375.545 = 579.399 N


2 2 2
40

c. Compute the direction angles measured from the positive reference axes
409.455  164.364 375.545
x = Arc cos y = Arc cos z = Arc cos
579.399 579.399 579.399
= 45.03 = 106.48 = 49.6

7. Three cable tensions T1, T2, and T3 act at the top of the flagpole. Given that the resultant force for the three
tensions is R = 500k N, find the magnitude of each of the cable tensions. (Pytel, 1996)
z

A
A
T3
T1
T2
12 m

3m D
8m 4m
B D y
O

B 6m
C
x C

Solution:

Force Position Vector x, m y, m z, m d, m


T1 AB 0 –8 –12 14.422
T2 AC 6 0 –12 13.416
T3 AD –4 3 –12 13

a. Write the tensions in Cartesian vector form


 8 j  12k
T1  T1 = 0.555T1 j  0.832T1k
14.422
6i  12k
T2  T2 = 0.447T2 i  0.894T2k
13.416
 4i  3 j  12k
T3  T3 = 0.308T3 i + 0.231T3 j  0. 923T3k
13
b. Equate the resultant force with the sum of the three forces
R= T1 + T2 + T3
–500k = 0.555T1 j  0.832T1k + 0.447T2 i  0.894T2 k  0.308T3 i + 0.231T3 j  0. 923T3k
–500k = (0.447T2  0.308T3)i + (0.555T1+ 0.231T3)j + ( 0.832T1  0.894T2  0. 923T3)k
c. Equate corresponding components
Rx = 0.447 T2 – 0.308 T3; 0 = 0.447 T2 – 0.308 T3 (1)
Ry =  0.555 T1 + 0.231 T3; 0=  0.555 T1 + 0.231 T3 (2)
Rz =  0.832 T1 – 0.894 T2 – 0.923 T3; 500 =  0.832 T1 – 0.894 T2 – 0.923 T3 (3)
d. Solving the three equations simultaneously; we get
T1 = 110.345 N; T2 = 182.759 N; and T3 = 265.252 N
41
EXERCISES
1. Determine the magnitude and direction of the resultant force. Ans. R = 551.222 N θ; θ = 44.44º
450 N
350 N y
1 y
2
45
x
275 N

2. The resultant of the three forces shown is directed vertically upward with a magnitude of 1200 N. Find the
magnitude and direction  of F. Ans. F = 909.882 N; θ = 15.92º
y F
750 N 500 N

3
30 4
x

3. Determine the magnitude and direction θ of F3 so that the resultant force R is directed along the positive x′
axis and has a magnitude of 600 N. Ans. F3 = 265.953 N; θ = 34.33º
y

x'
F1 = 450 N

30
x
F2 = 300 N
θ

30
F3

4. Boom AB is held in the position shown by three cables. Knowing that the tensions in cables AC and AD are
900 N and 1200 N, respectively, determine the tension in cable AE if the resultant of the tensions exerted at
point A of the boom must be directed along AB. What is the corresponding magnitude of the resultant force?
(Beer, 2007) Ans. FAE = 1659.446 N ; R = 2064.651 N
C A
30

D
65 50 E
B
42
5. In the figure shown, determine the required tension in cable AC, knowing that the resultant of the three
forces exerted at point C of boom BC must be directed along BC, and also determine the corresponding
magnitude of the resultant. (Beer, 2010) Ans. FAC = 95.075 lb; R = 94.967 lb

65º C

25º
35º
50 lb 75 lb

6. Determine the magnitude and the coordinate direction angles of the resultant force. (Riley, 1993)
Ans. 3264.334 N; θx = 72.03º, θy = 42.83º, θz = 52.72º

2m 3m

F3 = 1500 N
4m

F2 = 1200 N
y
2m
F1 = 1000 N
2m

7. Find the magnitudes of the three forces shown, given that their resultant force is R = {200i 100j + 50k}
kN. (Pytel, 1996) Ans. P1 = 83.333 kN; P2 = – 155.552 kN; P3 = 290.96 kN
z

2m

2m

P2
P3
3m y
P1

3m
x
43
Worksheet No. 5 – Resultant of Concurrent Forces

NAME: _______________________________ DATE: ___________________

Solve each of the following problems.

1. Determine the magnitude and direction of the resultant of the three forces shown.

y 400 N
575 N 750 N
25

45 45
x

2. If the magnitude of the resultant force acting on the eyebolt is 500 N and its direction measured
counterclockwise from the positive x axis is 30º, determine the magnitude of F1 and the angle θ.

y
F1 y
650 N

70
x
25

875 N
44
3. Determine the resultant of the two tensile forces at B if the tension in cable BC = 1350 N and the tension in
cable BD = 950 N.
z

2m
C D
4m
2m

6m
O 6m B
y

3m
x
A

4. The resultant of the three forces acting at A is 675 lb directed downward. Find the tension in AB, AC, and
AD.
y

A
A

12 ft
B
3 ft
6 ft
4 ft O x
6 ft 4 ft
C D

z
45

Chapter 3 – EQUILIBRIUM OF PARTICLES

Objectives:
At the end of the chapter, the students should be able to:
1. know the conditions of equilibrium for a particle, both coplanar and non-coplanar.
2. understand and use the concept of a free body to analyze equilibrium problems.
3. solve problems on equilibrium of particles in two dimensions and three dimensions.

3.a. Conditions for Equilibrium of a Particle


Equilibrium – it is the term used to designate the condition where the resultant of a force system is zero. A
particle is said to be in equilibrium if it remains at rest if originally at rest, or has a constant
velocity if in motion.
To maintain equilibrium, it is necessary to satisfy Newton’s first law of motion, which requires the
resultant force acting on a particle to be equal to zero. This condition, in mathematical form is
R = F = 0
where F is the vector sum of all forces acting on the particle.

3.b. Coplanar Force Systems


If a particle is subjected to a system of coplanar forces that lie in the x-y plane, the forces can be
resolved into i and j components. For equilibrium, these forces must have a sum of zero, i. e.,
F = 0 or Fxi + Fyj = 0
In scalar form, this equation can be written as
Fx = 0 and Fy = 0
These two equations mean that the sum of the rectangular components of the force system in any
direction must be zero. These equations can be used to determine two unknown quantities.

3.c. Free-body Diagram


Free-body diagram (FBD) – a drawing that shows the particle with all the forces, known and unknown, that act
on it.
In drawing a free-body diagram of a body, certain assumptions are made regarding the nature of the
forces (reactions) exerted by other bodies on the body of interest. Three common assumptions are the following:

1. Springs
The magnitude of the force exerted on a linearly elastic spring is
F = ks
ℓo where: k = stiffness or spring constant (a characteristic that defines the elasticity of a

spring
s s = deformation (elongation or compression) of the spring, measured from its
unloaded position,
= l – lo, where l is the stretched length and lo is the undeformed length
F

2. Cables and Pulleys


If s is positive, causing an elongation, then F must pull on the spring; if s is
Cables (or the like) can support only a tension or pulling force, and this force
negative, causing
always acts in thea direction
shortening, then
of the F must
cable. It ispush on it.that a cable has a negligible
assumed
weight and it cannot stretch.
The tension force developed in a continuous cable which passes over a
frictionless pulley must have a constant magnitude to keep the cable in equilibrium.
Hence, for any angle θ, the cable is subjected to a constant tension T throughout its
length.
46
3. Smooth or Frictionless Surfaces

If a surface of contact at which a force is applied by one body to


another body has only a small degree of roughness, it may be assumed to be
smooth (frictionless), therefore the action (or reaction) of the body on the
90 other is directed normal to the surface of contact.

R
When applying each of the two equations of equilibrium, we must account for the sense of direction of
any component by using an algebraic sign which corresponds to the arrowhead direction of the component along
the x or y axis. It is important to note that if a force has an unknown magnitude, then the arrowhead sense of the
force on the free-body diagram can be assumed. Then if the solution yields a negative scalar, this indicates that
the sense of the force is opposite to that which was assumed.

SAMPLE PROBLEMS
1. A force P is supported by two cables that are tied together at C as shown. Knowing that the maximum
allowable tension is 800 N in cable AC and 1500 N in cable BC, determine the magnitude of the largest
force P and the corresponding value of .

35 50
B
A 

C

P
Solution:
y
a. Draw the FBD of joint C.
BC = 1500 N
AC = 800 N
35 50
 x
 C
P
P
b. Apply the equations of equilibrium to find the components of P
Fx = 0] –Px + 1500 cos 50 – 800 cos 35 = 0; Px = 308.86 N
Fy = 0] Py + 1500 sin 50 + 800 sin 35 = 0; Py = 1607.928 N
c. Determine the magnitude and direction angle of P

P= 308.86 2  1607.928 2 = 1637.323 N


1607.928
α  Arc tan = 79.13
308.86
2. Four forces act on the particle shown. Determine the magnitude and direction angle  of force F4 for
equilibrium of the particle.
y

F2 = 600 N F3 = 1000 N

75 60
x
45  
F1 = 750 N F4
47
Solution:
a. Find the components of the forces

Force x component, N y component, N


F1 = 750 N –750 cos 45º = –530.330 –750 sin 45º = –530.330
F2 = 600 N – 600 cos 75º = –155.291 600 sin 75º = 579.555
F3 = 1000 N 1000 cos 60º = 500 1000 sin 60º = 866.025
F4 F4x –F4y

b. Apply the equations of equilibrium to find the components of the unknown force
Fx = 0] –530.330 –155.291 + 500 + F4x = 0
F4x = 185.621 N
Fy = 0] –530.330 + 579.555 + 866.025 – F4y = 0
F4y = 915.250 N
c. Determine the magnitude and direction of F4
F4 = 185.621  915.250 = 933.883 N
2 2

915.250
θ  Arc tan = 78.54
185.621

3. A body with a mass of 250 kg is supported by the flexible cable system shown in the figure. Determine the
tensions in cables A, B, C, and D. (Riley, 1993)

A B
40 30

C
60
D

Solution:
a. Compute for the weight of the body
W = mg; W = 250 (9.81) = 2452.5 N
b. Draw the FBD of the joints where the cables intersect
y
y TC
x
TD 60 TA 20 30 TB
x 40 30
60
30
2452.5 N TC
Lower joint Upper joint
c. Apply the equation of equilibrium to the FBD of lower joint
Fy = 0] TC sin 60 – 2452.5 = 0; TC = 2831.903 N
Fx = 0] TC cos 60 – TD = 0; TD = 1415.952 N
48
d. Apply the equation of equilibrium to the FBD of upper joint
Fy = 0] TA cos 20  TC sin 30 = 0; TA = 1506.824 N
Fx = 0] TB  TA sin 20  TC cos 30 = 0; TB = 2967.864 N

4. Find the smallest value of P for which the crate shown will be in equilibrium in the position shown. (Pytel,
1996)
C

A 60 B
30
P

180 kg
Solution:
a. Compute for the weight of the crate
and draw the FBD of the joint B
W = mg; W = 180 (9.81) = 1765.8 N
y
T1

60
x
T2 30
P
1765.8 N
b. Apply the equation of equilibrium
Fy = 0] T1 sin 60  P sin 30  1765.8 = 0
0.866T1 – 0.5P = 1765.8 Eq. 1
Fx = 0] P cos 30  T1 cos 60  T2 = 0
For minimum P, T2 = 0
0.866P  0.5T1 = 0 Eq. 2
Solving the two equations simultaneously, we get P = 1765.342 N

5. Determine the stretch in each spring for equilibrium of the 2-kg block. The springs are shown in their
equilibrium position. (Hibbeler, 1986)
3m 4m

C
kAC = 20 B
3m N/m kAB = 30
A N/m
kAD= 40
N/m
D

Solution:
a. Compute for the stretch of spring AD:
FAD = 2 (9.81)
ks = 19.62
40s = 19.62; s = 0.4905 m
49
b. Draw the free body diagram of joint A and
apply the equations of equilibrium
y Fx = 0] 4 1
FAB  FAC  0 Eq. 1
FAC
FAB
5 2
1 3
1
4
3 Fy = 0] FAC  FAB  FAD  0
1 x 2 5
1 3 Eq. 2
FAC  FAB  19.62
2 5
FAD
Solving the two equations simultaneously, we get FAB = 14.014 N and FAC = 15.855 N.
c. Compute for the elongation of the springs
FAB = 14.014 FAC = 15.855
ks = 14.014 ks = 15.855
30s = 14.014; s = 0.467 m 20s = 15.855; s = 0.793 m

6. A homogeneous sphere with a weight of 75 lb rests on two smooth planes as shown in the figure. Determine
the forces exerted on the sphere by the planes at contact points A and B.

60 20
A
B
Solution: 75 lb
a. The forces acting on the sphere are the weight and the
reactions at the smooth planes. Draw the free body
diagram of the sphere.
b. Apply the equations of equilibrium
Fx = 0] RA sin 60º  RB sin 20º = 0 (1)
Fy = 0] RA cos 60º + RB cos 20º  75 = 0 (2) 60 20
RA RB
c. Solving the two equations simultaneously, we get
RA = 26.047 lb and RB = 65.954 lb

Alternative solution (using the force triangle):


Since there are only three forces acting on the sphere, the three forces form a triangle (force polygon
formed by the three forces that are in equilibrium).
a. Draw the force triangle
b. Apply sine law to find the required reactions
RA RB 75 RB
  20
sin 20 sin 60 sin 100
75 lb
75 sin 20 100
RA = = 26.047 lb
sin 100 60
RA
RB = 75 sin 60 = 65.954 lb
sin 100
50
7. The 200-mm diameter pipes shown each have a mass of 200 kg. Determine the force exerted by the supports
on the pipes at contact surfaces A, B, and C. Assume all surfaces to be smooth. (Riley, 1993)

C
B

45 A
Solution:
a. Find the weight of the cylinders
W = mg
= 200 (9.81)
= 1962 N

b. Draw the FBD of the pipes


R1 y
1962 N x 45
y 1962 N
45
 RB
x

RC R1 RA
45 45 45

Upper pipe Lower pipe

c. Apply the equations of equilibrium to the FBD of the upper pipe


Fx = 0] RC – 1962 cos 45 = 0
RC = 1387.344 N
Fy = 0] R1 – 1962 sin 45 = 0
R1 = 1387.344 N
d. Apply the equations of equilibrium to the FBD of the lower pipe
Fy = 0] RA cos 45  R1 cos 45  1962 = 0
RA = 4162.031 N
Fx = 0] R1 sin 45 + RA sin 45  RB = 0
RB = 3924 N

EXERCISES
1. Determine the magnitude and direction angle  of force F4 so that the particle shown is in equilibrium.
Ans. F = 1404.917 N; θ = 7.05º
y

F1 = 700 N
F4
F2 = 550 N 40
25 
x
60 

F3 = 1200 N
51
2. Find the force in each supporting cable if the crate has a weight of 850 lb. (Hibbeler, 2010)
Ans. FAB = 478 lb; FAC = 518 lb
C
B

30 45
A
D

3. If the mass of cylinder C is 40 kg, determine the mass of cylinder A in order to hold the assembly in the
position shown. (Hibbeler, 2010) Ans. mA = 20 kg

B
E 25
D
C
50 kg
A

4. If block D weighs 300 lb and block B weighs 275 lb, determine the required weight of block C and the angle
θ for equilibrium. (Hibbeler, 2010) Ans. WC = 240 lb; θ = 40.9°

θ 30
A

C
B D

5. Determine the maximum weight of the bucket that the wire system can support so that no single wire
develops a tension exceeding 100 lb. (Hibbeler, 2010) Ans. W = 57.7 lb
C

B 35
A D
4
3 35
E

D
6. The block has a mass of 5 kg and rests on the smooth plane. Determine the unstretched length of the spring
if the spring constant k = 200 N/m. (Hibbeler, 2010) Ans. lo = 0.283 m

0.3 m

0.4 m

45º
52
7. The unstretched length of spring AB is 3 m. If the block is held in the equilibrium position shown, determine
the mass of the block at D. (Hibbeler, 2010) Ans. m = 8.56 kg
3m 4m

C B

3m
kAC = 20 kAB = 30
N/m N/m
A

8. Two 10-in diameter pipes and a 6-in diameter pipe are supported in a pipe rack as shown in the figure. The
10-in diameter pipes each weigh 300 lb and the 6-in diameter pipe weighs 175 lb. Determine the forces
exerted on the pipes by the supports at contact surfaces A, B, and C. Assume all surfaces to be smooth.
(Riley, 1993) Ans. RA = 558.486 lb; RB = 403.083 lb; RC = 961.486 lb

14.5 in

9. The mass of cylinder A is 50 kg with a diameter of 200 mm. Cylinder B has a mass of 100 kg and a diameter
of 300 mm. Determine the forces exerted on the cylinders by the inclined surfaces and the magnitude and
direction of the force exerted by cylinder A on cylinder B when the cylinders are in equilibrium. Assume all
surfaces to be smooth. Ans. RA = 439.772 N; RB = 1201.47 N; R = 359.075 N θ ; θ = 30º

A
B

45º
15º
53
3.d. Three-dimensional Force Systems
For a three-dimensional system of concurrent forces, the conditions of equilibrium are satisfied only if
R = Rx î + Ry ĵ + Rz k̂ =0
In scalar form, this equation means that
Rx = Fx ; Fx = 0
Ry = Fy ; Fy = 0
Rz = Fz ; Fz = 0
These three equations state that the algebraic sum of the components of all the forces acting on the particle
along each of the coordinate axes must be zero. Using them we can solve for at most three unknowns. If the
solution for a force yields a negative result, this indicates that its sense is the reverse of that shown on the free-
body diagram.

SAMPLE PROBLEMS
1. The particle shown is in equilibrium under the action of the four forces on the free-body diagram. Determine
the magnitude and the coordinate direction angles of the unknown force F4.
z
F4
F3 = 300 lb

4 ft 30

F1 = 120 lb
3 ft y
x
F2 =250 lb
Solution:
a. Express the forces in Cartesian vector form
4 3
F1 = (120)î  (120)ĵ = 96î  72ĵ
5 5
F2 = 250 k̂
F3 = 300 cos 30ĵ + 300 sin 30 k̂ = 259.808ĵ + 150 k̂
F4 = F4x î + F4y ĵ + F4z k̂
b. Apply the equilibrium equations to find the components of F4
Fx = 0] 96 + F4x = 0; F4x = 96 lb
Fy = 0] 72 + 259.808 + F4y = 0; F4y = 187.808 lb
Fz = 0] 250 + 150 + F4z = 0; F4z = 100 lb
c. Find the magnitude and direction angles of the unknown force
F4  962  187.8082  1002 = 233.426 lb
 96
x = Arc cos = 114.28
233.426
 187.808
y = Arc cos = 143.57
233.426
100
z = Arc cos = 64.63
233.426
54
2. A 3000-lb cylinder is supported by a system of cables as shown in the figure. Determine the tensions in
cables A, B, and C. (Riley. 1993)
z
B

A 4 ft

6 ft
6 ft
4 ft
5 ft C
3 ft 4 ft
y
x
Solution:
a. Draw the free body diagram of the cylinder
TB
TC

TA

b. Express the forces in Cartesian vector form


4i  3 j  6k
TA = TA = 0.512 TA i  0.384TA j + 0.768TA k
4 2  32  6 2
 4i  6 j  4k
TB = TB = 0.485 TB i  0.728TB j + 0.485TB k
42  62  42
TC = TC j
W = 3000k
c. Apply the equilibrium equations
Fx = 0] 0.512 TA  0.485 TB = 0 (1)
Fy = 0]  0.384TA  0.728TB + TC = 0 (2)
Fz = 0] 0.768TA + 0.485TB 3000 = 0 (3)
e. Solve the three equations simultaneously to get
TA = 2343.487 lb
TB = 2474.643 lb
TC = 2701.439 lb
55
3. A container of weight W is suspended from ring A to which cables AC and AE are attached. A force P is
applied to the end F of a third cable which passes over a pulley at B and through ring A and which is
attached to a support at D. Knowing that W = 1000 N, determine the magnitude of P. (Beer, 2010)
y

0.86 m 0.4 m

1.2 m
E
B 1.3 m

O
0.78 m C
D 0.4 m x

F
z 1.6 m A
P

Solution: TAE
a. Draw the free-body diagram of joint A P
TAC P

W = 1800 N
b. Write the forces in Cartesian vector form
 0.78 î  1.6 ĵ
TAB = P = 0.438P î + 0.899P ĵ
0.78 2  1.6 2
1.6ˆj  1.2kˆ
TAC = TAC = 0.8TAC ĵ + 0.6TAC k̂
1.6 2  1.2 2
1.3î  1.6 ĵ  0.4k̂
TAD = P = 0.619P î + 0.762P ĵ + 0.19P k̂
1.32  1.6 2  0.4 2
 0.4ˆi  1.6ˆj  0.86kˆ
TAE = TAE = 0.215TAE î + 0.86TAE ĵ  0.462TAE k̂
0.4 2  1.6 2  0.86 2
W = 1000 ĵ

c. Apply the equilibrium equations


Fx = 0] 0.438P + 0.619P  0.215AE = 0
0.181P  0.215AE = 0 (1)
Fy = 0] 0.899P + 0.8AC + 0.762P + 0.86AE 1000 = 0
1.661P + 0.8AC + 0.86AE = 1000 (2)
Fz = 0] 0.6AC + 0.19P  0.462AE = 0 (3)

d. Solving for P from the three equations, we get P = 377.256 N


56
4. A tripod supports a load P of unknown magnitude. Find the maximum magnitude of P that can be supported
if the force in any leg is limited to 8 kN. Assume that the legs are two-force bodies. (Pytel, 1996)
z

0.9 m 1.8 m
2.4 m C
y
1.8 m
B
O
1.8 m
D

x
Solution:
a. Draw the free-body diagram of joint A
P

FD FC FB
b. Write the forces in Cartesian vector form
 1.8 ĵ  2.4k̂
FB = FB = 0.6FB ĵ + 0.8FB k̂
1.8 2  2.4 2
1.8 î  0.9 ĵ  2.4k̂
FC = FC = 0.575FC î + 0.287FC ĵ + 0.766FC k̂
1.8 2  0.9 2  2.4 2
 1.8 î  0.9 ĵ  2.4k̂
FD = FD = 0.575FD î + 0.287FD ĵ + 0.766FD k̂
1.4 2  0.9 2  2.4 2
P = P k̂

c. Apply the equilibrium equations


Fx = 0] 0.575FC  0.575FD = 0 ; FC = FD (1)
Fy = 0] 0.6FB + 0.287FC + 0.287FD = 0 (2)
Fz = 0] 0.8FB + 0.766FC + 0.766FD P = 0 (3)
If FD = 8 kN, then, from equation (1), FC = 8 kN and from equation (2), FB = 7.653 kN, hence,
the legs are not overloaded. Solving for P from equation (3), we get P = 18.379 kN.
57
5. The light boom AB is attached to the vertical wall by a ball-and-socket joint at A and supported by two
cables at B. A force P = {18i  24k} kN is applied at B. Compute the cable tensions and the reaction at A.
z
2m
C D
4m 2m

6m
O 6m B
y

3m
x P

Solution:
a. Draw the free-body diagram of joint B. The total reaction at A is equal to the force along member
AB.
FBD
FBC
B

P
A

RA

b. Write the tensions in Cartesian vector form


2 î  6 ĵ  6k̂
TBC = TBC = 0.229 TBC î  0.688TBC ĵ + 0.688TBC k̂
22  62  62
 4 î  6 ĵ  2k̂
TBD = TBD = 0.535 TBD î  0.802TBD ĵ + 0.267TBD k̂
42  62  22
 2 î  6 ĵ  3k̂
RA = R A = 0.286RA î + 0.857RA ĵ + 0.429RA k̂
2 2  6 2  32
P = 18î 24 k̂

c. Apply the equilibrium equations


Fx = 0] 0.229 TBC  0.535 TBD  0.286RA +18 = 0 (1)
Fy = 0]  0.688TBC  0.802TBD + 0.857RA = 0 (2)
Fz = 0] 0.688TBC + 0.267TBD + 0.429RA  24 = 0 (3)

d. Solving the three equations simultaneously, we get


TBC = 8.731 kN, TBD = 22.419 kN, and RA = 27.989 kN
58
6. The 500-lb crate is suspended from the cable system shown in the figure. Determine the force in each
segment of the cable; i.e., AB, AC, and CD, and the force in cables CE and CF. (Hibbeler, 1986)

10 ft
z

y
24 ft
B
C x

A 20º
35º

24 ft 500 lb
E
7 ft
7 ft
F
Solution:
a. Write the tensions in Cartesian vector form
TAB = TAB cos 20 î + TAB sin 20 k̂ = 0.94TAB î + 0.342TAB k̂
TAC = TAC cos 35 î + TAC sin 35 k̂ =  0.819TAC î + 0.574TAC k̂
 10 î  24k̂
TCD = TCD =  0.385TCD î + 0.923TCD k̂
10 2  24 2
7 ĵ  24k̂
TCE = TCE = 0.28TCE ĵ  0.96TCE k̂
7 2  24 2
 7 ĵ  24k̂
TCF = TCF =  0.28TCF ĵ  0.96TCF k̂
7 2  24 2

b. FBD of Joint A
z Fx = 0] 0.94TAB  0.819TAC = 0 (1)
TAB
TAC Fz = 0] 0.342TAB + 0.574TAC  500 = 0 (2)
A 20º x
35º Solving the two equations simultaneously, we get
TAB = 499.599 lb and TAC = 573.410 lb.

500 lb
c. FBD of Joint C
z Fx = 0] 0.385TCD + 0.819TAC = 0
TCD = 1219.799 lb
TCD Fy = 0] 0.28TCE  0.28TCF = 0
y
TCE =TCF (3)

C
x Fz = 0] 0.923TCD  0.96TCE  0.96TCF  0.574TAC = 0
35º TCE + TCF = 829.935 (4)
TAC Solving the two equations simultaneously, we get
TCE = TCF = 414.968 lb
TCE TCF
59
EXERCISES

1. Determine the tension developed in cables AB, AC, and AD. (Hibbeler, 2010) Ans. FAB =506.25 N,
FAC =843.75 N, FAD =1125 N
z
D

4
3
C
3
A
4 y
B
x 900 N

2. Determine the maximum weight of the crate so that the tension developed in any cable does not exceed 450
lb. (Hibbeler, 2010) Ans. W = 375 lb
z
C B
2 ft
1 ft 2 ft 1 ft

2 ft 2 ft
y
A
3 ft

D
x

3. The joint of a space frame is subjected to four member forces. Member OA lies in the plane x–y and member
OB lies in the plane y–z. Determine the forces acting in each of the members required for equilibrium of the
joint. (Hibbeler, 2010) Ans. F1 = 0, F2 = 311 lb, F3 = 238 lb
z
y

4. A container of weight W is suspended from ring A. Cable BAC passes through the ring and is attached to
fixed supports at B and C. Two forces P = Pi and Q = Qk are applied to the ring to maintain the container in
the position shown. Knowing that W = 376 N, determine P and Q. (Beer, 2010) Ans. P = 131.2 N and
Q = 29.6 N
y

150 mm
160 mm
C
B
130 mm
240 mm
z x
400 mm
A
P
Q
A
W
60
Worksheet No. 6 – Equilibrium of Particles

NAME: _______________________________ DATE: ___________________

Solve the given problems.

1. An irregularly shaped machine component is held in the position shown by three clamps. Knowing that
FA = 940 N, determine the magnitudes of the forces FB and FC exerted by the other two clamps.
FB

FA

FC

2. A homogeneous steel cylinder weighing 500 lb is supported by a flexible cable and a smooth inclined plane
as shown in the figure. Determine the tension T in the cable and the force R exerted by the inclined plane on
the cylinder.

70º

60º
61
3. Determine the force P needed to hold the cable in the position shown. Also, compute the sag yB and the
maximum tension in the cable.

A E
3m
yB D
B C

6 kN
4 kN P
4m 6m 3m 2m

4. The support assembly shown is bolted in place at B, C, and D and supports a downward force P at A.
Knowing that the forces in members AB, AC, and AD are directed along the respective members and that the
force in member AB is 29.2 N, determine the magnitude of P.

You might also like